K
Khách

Hãy nhập câu hỏi của bạn vào đây, nếu là tài khoản VIP, bạn sẽ được ưu tiên trả lời.

NV
28 tháng 3 2023

Chắc đề là \(x+y+z=3\)

Ta có: 

\(\left(2x+y+z\right)^2=\left(x+y+x+z\right)^2\ge4\left(x+y\right)\left(x+z\right)\)

\(\Rightarrow P\le\dfrac{x}{4\left(x+y\right)\left(x+z\right)}+\dfrac{y}{4\left(x+y\right)\left(y+z\right)}+\dfrac{z}{4\left(x+z\right)\left(y+z\right)}\)

\(\Rightarrow P\le\dfrac{x\left(y+z\right)+y\left(z+x\right)+z\left(x+y\right)}{4\left(x+y\right)\left(y+z\right)\left(z+x\right)}=\dfrac{xy+yz+zx}{2\left(x+y\right)\left(y+z\right)\left(z+x\right)}\)

Mặt khác:

\(\left(x+y\right)\left(y+z\right)\left(z+x\right)=\left(xy+yz+zx\right)\left(x+y+z\right)-xyz\)

\(=\left(x+y+z\right)\left(xy+yz+zx\right)-\sqrt[3]{xyz}.\sqrt[3]{xy.yz.zx}\)

\(\ge\left(x+y+z\right)\left(xy+yz+zx\right)-\dfrac{1}{3}.\left(x+y+z\right).\dfrac{1}{3}\left(xy+yz+zx\right)\)

\(=\dfrac{8}{9}\left(x+y+z\right)\left(zy+yz+zx\right)=\dfrac{8}{3}\left(xy+yz+zx\right)\)

\(\Rightarrow P\le\dfrac{xy+yz+zx}{2.\dfrac{8}{3}\left(xy+yz+zx\right)}=\dfrac{3}{16}\)

Dấu "=" xảy ra khi \(x=y=z=1\)

27 tháng 10 2020

Bài 2: Ta có: x, y, z không âm và \(x+y+z=\frac{3}{2}\)nên \(0\le x\le\frac{3}{2}\Rightarrow2-x>0\)

Áp dụng bất đẳng thức AM - GM dạng \(ab\le\frac{\left(a+b\right)^2}{4}\), ta được: \(x+2xy+4xyz=x+4xy\left(z+\frac{1}{2}\right)\le x+4x.\frac{\left(y+z+\frac{1}{2}\right)^2}{4}=x+x\left(2-x\right)^2\)

Ta cần chứng minh \(x+x\left(2-x\right)^2\le2\Leftrightarrow\left(2-x\right)\left(x-1\right)^2\ge0\)*đúng*

Đẳng thức xảy ra khi \(\left(x,y,z\right)=\left(1,\frac{1}{2},0\right)\)

29 tháng 10 2020

Bài 3: Áp dụng đánh giá quen thuộc \(4ab\le\left(a+b\right)^2\), ta có: \(2\le\left(x+y\right)^3+4xy\le\left(x+y\right)^3+\left(x+y\right)^2\)

Đặt x + y = t thì ta được: \(t^3+t^2-2\ge0\Leftrightarrow\left(t-1\right)\left(t^2+2t+2\right)\ge0\Rightarrow t\ge1\)(dễ thấy \(t^2+2t+2>0\forall t\))

\(\Rightarrow x^2+y^2\ge\frac{\left(x+y\right)^2}{2}\ge\frac{1}{2}\)

\(P=3\left(x^4+y^4+x^2y^2\right)-2\left(x^2+y^2\right)+1=3\left[\frac{3}{4}\left(x^2+y^2\right)^2+\frac{1}{4}\left(x^2-y^2\right)^2\right]-2\left(x^2+y^2\right)+1\ge\frac{9}{4}\left(x^2+y^2\right)^2-2\left(x^2+y^2\right)+1\)\(=\frac{9}{4}\left[\left(x^2+y^2\right)^2+\frac{1}{4}\right]-2\left(x^2+y^2\right)+\frac{7}{16}\ge\frac{9}{4}.2\sqrt{\left(x^2+y^2\right)^2.\frac{1}{4}}-2\left(x^2+y^2\right)+\frac{7}{16}=\frac{9}{4}\left(x^2+y^2\right)-2\left(x^2+y^2\right)+\frac{7}{16}=\frac{1}{4}\left(x^2+y^2\right)+\frac{7}{16}\ge\frac{1}{8}+\frac{7}{16}=\frac{9}{16}\)Đẳng thức xảy ra khi x = y = 1/2

10 tháng 8 2017

post từng câu một thôi bn nhìn mệt quá

25 tháng 7 2017

bài này cần x,y,z>0 nữa, vừa xem xong bài y hệt của LCC :v

Dự đoán dấu "=" khi \(x=y=z=1\) thì \(P=24\)

Ta chứng minh P=24 là GTNN

Thật vậy áp dụng BĐT C-S ta có:

\(P=Σ\frac{\left(x+1\right)^2\left(y+1\right)^2\left(z+1\right)^2}{\left(z^2+1\right)\left(x+y\right)^2}\ge\frac{\left(Σ\left(x+1\right)\left(y+1\right)\left(x+y\right)\right)^2}{Σ\left(z^2+1\right)\left(x+y\right)^2}\)

Cần chứng minh: \(\frac{\left(Σ\left(x+1\right)\left(y+1\right)\left(x+y\right)\right)^2}{Σ\left(z^2+1\right)\left(x+y\right)^2}\ge24\)

\(\Leftrightarrow\left(Σ\left(x+1\right)\left(y+1\right)\left(x+y\right)\right)^2\ge24Σ\left(z^2+1\right)\left(x+y\right)^2\)

Đặt \(\hept{\begin{cases}x+y+z=3u\\xy+yz+xz=3v^2\\xyz=w^3\end{cases}}\) \(\Rightarrow u=1\) thì

\(Σ\left(x+1\right)\left(y+1\right)\left(z+1\right)=Σ\left(x^2y+x^2z+2x^2+2xy+2x\right)\)

\(=9uv^2-3w^3+2u\left(9u^2-6v^2\right)+9uv^2+6u^3=3\left(8u^3+uv^2-w^3\right)\)

Và  \(Σ\left(z^2+1\right)\left(x+y\right)^2=2Σ\left(x^2y^2+x^2yz+x^2u+xyu^2\right)\)

\(=2\left(9v^4-6uw^3+3uw^3+9u^4-6u^2v^2+3u^2v^2\right)\)

\(=6\left(3u^4-u^2v^2+3v^4-uw^3\right)\). Can cm \(f\left(w^3\right)\ge0\)

\(f\left(w^3\right)=\left(8u^3+uv^2-w^3\right)^2-16\left(3u^6-u^4v^2+3u^2v^4-u^3w^3\right)\)

\(f'\left(w^3\right)=-2\left(8u^3+uv^2-w^3\right)+16u^3=2w^3-2uv^2\le0\)

Thay \(f\) la ham` ngh!ch bien, do đó, BĐT có 1 GTLN của w3 khi 2 biến bằng nhau

Đặt \(y=x;z=3-2x\), Khi đó: 

\(BDT\Leftrightarrow\left(x-1\right)^2\left(x^4-2x^3-11x^2+24x+4\right)\ge0\)

14 tháng 1 2021

Áp dụng bất đẳng thức AM - GM:

\(P\ge3\sqrt[3]{\dfrac{\left(xy+1\right)\left(yz+1\right)\left(zx+1\right)}{xyz}}\).

Áp dụng bất đẳng thức AM - GM ta có:

\(xy+1=xy+\dfrac{1}{4}+\dfrac{1}{4}+\dfrac{1}{4}+\dfrac{1}{4}\ge5\sqrt[5]{\dfrac{xy}{4^4}}\).

Tương tự: \(yz+1\ge5\sqrt[5]{\dfrac{yz}{4^4}};zx+1\ge5\sqrt[5]{\dfrac{zx}{4^4}}\).

Do đó \(\left(xy+1\right)\left(yz+1\right)\left(zx+1\right)\ge125\sqrt[5]{\dfrac{\left(xyz\right)^2}{4^{12}}}\)

\(\Rightarrow\dfrac{\left(xy+1\right)\left(yz+1\right)\left(zx+1\right)}{xyz}\ge125\sqrt[5]{\dfrac{1}{4^{12}\left(xyz\right)^3}}\).

Mà \(xyz\le\dfrac{\left(x+y+z\right)^3}{27}=\dfrac{1}{8}\)

Nên \(\dfrac{\left(xy+1\right)\left(yz+1\right)\left(zx+1\right)}{xyz}\ge125\sqrt[5]{\dfrac{8^3}{4^{12}}}=125\sqrt[5]{\dfrac{1}{2^{15}}}=\dfrac{125}{8}\)

\(\Rightarrow P\ge\dfrac{15}{2}\).

Vậy...

 

 

 

17 tháng 1 2021

Áp dụng bất đẳng thức AM - GM:

P≥33√(xy+1)(yz+1)(zx+1)xyz.

Áp dụng bất đẳng thức AM - GM ta có:

xy+1=xy+14+14+14+14≥55√xy44.

Tương tự: yz+1≥55√yz44;zx+1≥55√zx44.

Do đó (xy+1)(yz+1)(zx+1)≥1255√(xyz)2412

⇒(xy+1)(yz+1)(zx+1)xyz≥1255√1412(xyz)3.

Mà xyz≤(x+y+z)327=18

Nên  (xy+1)(yz+1)(zx+1)xyz≥1255√83412=1255√1215=1258 

⇒P≥152.